Prove that the following map is compactCompact operator and compact embeddingsShowing that smoothing...

What is the orbit and expected lifetime of Crew Dragon trunk?

Sort array by month and year

How to install "rounded" brake pads

Is divide-by-zero a security vulnerability?

A running toilet that stops itself

What does it take to become a wilderness skills guide as a business?

What can I do if someone tampers with my SSH public key?

Giving a career talk in my old university, how prominently should I tell students my salary?

If nine coins are tossed, what is the probability that the number of heads is even?

Will the concrete slab in a partially heated shed conduct a lot of heat to the unconditioned area?

Why does this boat have a landing pad? (SpaceX's GO Searcher) Any plans for propulsive capsule landings?

I can't die. Who am I?

Who has more? Ireland or Iceland?

What should I do when a paper is published similar to my PhD thesis without citation?

What does "rhumatis" mean?

How spaceships determine each other's mass in space?

Ultrafilters as a double dual

Why aren't there more Gauls like Obelix?

Does an unused member variable take up memory?

How can I portion out frozen cookie dough?

Use Mercury as quenching liquid for swords?

Can inspiration allow the Rogue to make a Sneak Attack?

Why does a car's steering wheel get lighter with increasing speed

Is it a Cyclops number? "Nobody" knows!



Prove that the following map is compact


Compact operator and compact embeddingsShowing that smoothing operators are compactArzela-Ascoli and adjoint of compact operator compactProof of equicontinuous and pointwise bounded implies compactEquicontinuous and pointwise bounded implies compactweakly continuous linear mapArgue, why these operators are compact or not.Show set of functions compact and convexShowing a set is compact using Arzelà–Ascoli theoremFor which n differential operator $C^{(n)}[0, 1] rightarrow C[0,1]$ is compact













1












$begingroup$


I'm studying by myself PDEs without having done Functional Analysis and I'm trying do the following exercise of the book "Partial Differential Equations I" by Michael E. Taylor on Appendix $A$ - Section $6$ (page $592$):





  1. Prove the following result, also known as part of Ascoli's theorem. If $X$ is a compact metric space, $B_j$ are Banach spaces, and $K: B_1 longrightarrow B_2$ is a compact operator, then $kappa f(x) = K(f(x))$ defines a compact map $kappa: mathcal{C}^{alpha}(X,B_1) longrightarrow C(X,B_2)$, for any $alpha > 0$.




$mathcal{C}^{alpha}$ denotes a space with $alpha$-Holder continuity (this notation was introduced on the final of the page $317$).



I would like to know if my attempt it's correct until where I wrote and, if it is correct, how I can ensure the limit below lives in $C(X,B_2)$.




$textbf{My attempt:}$



Firstly, we observe that $kappa$ is well-defined, otherwise, $K$ wouldn't well defined, which would be an absurd since $K$ is a map.



Now, given a bounded sequence $(f_n)$ in $mathcal{C}^{alpha}(X,B_1)$ and fixing $x in X$, we have that $(f_n(x))$ is a bounded sequence in $B_1$. By compactness of the map $K$, there is a subsequence $(f_{n_l}(x))$ in $B_1$ such that $K(f_{n_l}(x)) rightarrow K(f(x)) in B_2$, i.e., given $varepsilon > 0$, there is $N(x) in mathbb{N}$ such that



$$l > N(x) Longrightarrow ||K(f_{n_l}(x)) - K(f(x))||_{B_2} < frac{varepsilon }{3} (*)$$



The subsequence $((K circ f_{n_l}))$ is in $C(X,B_2)$ since $K$ and $f_{n_l}$ are continuous functions. Since the limit is unique, $f$ is well-defined.



By continuity of $K circ f_{n_l}$ in every $x in X$, there is $delta_x > 0$ such that



$$||y - x||_X < delta_x Longrightarrow ||K (f_{n_l})(y) - K (f_{n_l})(x)||_{B_2} < frac{varepsilon}{3} (**)$$



W.l.o.g., we assume $frac{1}{N(x)} < delta_x$. By compactness of $X$, there is a finite subcover of $X$ by open balls $B(x_i,frac{1}{N(x_i)}) subset X$, $i = 1, cdots, j$. Thus, arguing as before, we find $(K circ f_{n_l}(x))$ such that $(*)$ holds for $N := max_limits{ i in { 1,cdots,j } } N(x_i)$. Denoting by $delta := frac{1}{N}$ and using $(*)$ and $(**)$, we have that



$||K(f(y)) - K(f(x))||_{B_2} leq ||K(f(y)) - K(f_{n_l}(y))||_{B_2} + ||K(f_{n_l}(y)) - K(f_{n_l}(x))||_{B_2} + ||K(f_{n_l}(x)) - K(f(x))||_{B_2}$



$begin{eqnarray*}
||K(f(y)) - K(f(x))||_{B_2} &leq& ||K(f(y)) - K(f_{n_l}(y))||_{B_2} + ||K(f_{n_l}(y)) - K(f_{n_l}(x))||_{B_2} + ||K(f_{n_l}(x)) - K(f(x))||_{B_2}\
&leq& frac{varepsilon}{3} + frac{varepsilon}{3} + frac{varepsilon}{3} = varepsilon,
end{eqnarray*}$



whenever $l > N$ and $||y - x||_X < delta$, hence $(K circ f) in C(X,B_2)$. $square$




I'm stuck here, because I can't prove that $(K circ f) in C(X,B_2)$. I know that $(K circ f_{n_l}) in C(X,B_2)$ for each $l in mathbb{N}^*$, $(K circ f_{n_l})(x) rightarrow (K circ f)(x)$, $(K circ f_{n_l})$ is bounded I know the uniform limit of continuous functions is continuous, but I can't see how I can prove the pointwise convergence is, indeed, uniform. The only thing that I thought about this is use Ascoli's theorem because I know that $(K circ f_{n_l})$ is bounded, but I can't see how the family ${ (K circ f_{n_l}) in C(X,B_2) ; l in mathbb{N}^* }$ is equicontinuous.



I thought about the MaoWao's comment and I think I'm close to solving the question, but I don't sure if $(*)$ by the $N$ that I chose. If this is not the right $N$ that I need to take, someone can help me with a hint about how I can choose the $N$ without depending on $x$?



Thanks in advance!



$textbf{EDIT:}$



The version of Ascoli's theorem that I'm using:




$textbf{Ascoli-Arzela's theorem:}$ let be $E$ a set of continuous maps $f: K longrightarrow N$, where $K$ is compact. $E subset C(K,N)$ is relatively compact if, and only if, the following holds:



1) $E$ is equicontinuous;



2) For each $x in K$, $E(x) = { f(x) ; f in E }$ is relatively compact in $N$.











share|cite|improve this question











$endgroup$












  • $begingroup$
    The problem with your attempt is that the subsequence you choose depends on the point $x$. It is not immediate why there should be a subsequence working for all points simultaneously. Also, could you mention which version of Ascoli's theorem (if any) you already know.
    $endgroup$
    – MaoWao
    2 days ago












  • $begingroup$
    @MaoWao, I included the version of Ascoli's theorem that I have in mind.
    $endgroup$
    – George
    2 days ago










  • $begingroup$
    In your version of Ascoli-Arzela, what is $N$?
    $endgroup$
    – Nate Eldredge
    yesterday










  • $begingroup$
    @NateEldredge, $N$ is a metric space, this version of AScoli-Arzela is for metric spaces
    $endgroup$
    – George
    yesterday
















1












$begingroup$


I'm studying by myself PDEs without having done Functional Analysis and I'm trying do the following exercise of the book "Partial Differential Equations I" by Michael E. Taylor on Appendix $A$ - Section $6$ (page $592$):





  1. Prove the following result, also known as part of Ascoli's theorem. If $X$ is a compact metric space, $B_j$ are Banach spaces, and $K: B_1 longrightarrow B_2$ is a compact operator, then $kappa f(x) = K(f(x))$ defines a compact map $kappa: mathcal{C}^{alpha}(X,B_1) longrightarrow C(X,B_2)$, for any $alpha > 0$.




$mathcal{C}^{alpha}$ denotes a space with $alpha$-Holder continuity (this notation was introduced on the final of the page $317$).



I would like to know if my attempt it's correct until where I wrote and, if it is correct, how I can ensure the limit below lives in $C(X,B_2)$.




$textbf{My attempt:}$



Firstly, we observe that $kappa$ is well-defined, otherwise, $K$ wouldn't well defined, which would be an absurd since $K$ is a map.



Now, given a bounded sequence $(f_n)$ in $mathcal{C}^{alpha}(X,B_1)$ and fixing $x in X$, we have that $(f_n(x))$ is a bounded sequence in $B_1$. By compactness of the map $K$, there is a subsequence $(f_{n_l}(x))$ in $B_1$ such that $K(f_{n_l}(x)) rightarrow K(f(x)) in B_2$, i.e., given $varepsilon > 0$, there is $N(x) in mathbb{N}$ such that



$$l > N(x) Longrightarrow ||K(f_{n_l}(x)) - K(f(x))||_{B_2} < frac{varepsilon }{3} (*)$$



The subsequence $((K circ f_{n_l}))$ is in $C(X,B_2)$ since $K$ and $f_{n_l}$ are continuous functions. Since the limit is unique, $f$ is well-defined.



By continuity of $K circ f_{n_l}$ in every $x in X$, there is $delta_x > 0$ such that



$$||y - x||_X < delta_x Longrightarrow ||K (f_{n_l})(y) - K (f_{n_l})(x)||_{B_2} < frac{varepsilon}{3} (**)$$



W.l.o.g., we assume $frac{1}{N(x)} < delta_x$. By compactness of $X$, there is a finite subcover of $X$ by open balls $B(x_i,frac{1}{N(x_i)}) subset X$, $i = 1, cdots, j$. Thus, arguing as before, we find $(K circ f_{n_l}(x))$ such that $(*)$ holds for $N := max_limits{ i in { 1,cdots,j } } N(x_i)$. Denoting by $delta := frac{1}{N}$ and using $(*)$ and $(**)$, we have that



$||K(f(y)) - K(f(x))||_{B_2} leq ||K(f(y)) - K(f_{n_l}(y))||_{B_2} + ||K(f_{n_l}(y)) - K(f_{n_l}(x))||_{B_2} + ||K(f_{n_l}(x)) - K(f(x))||_{B_2}$



$begin{eqnarray*}
||K(f(y)) - K(f(x))||_{B_2} &leq& ||K(f(y)) - K(f_{n_l}(y))||_{B_2} + ||K(f_{n_l}(y)) - K(f_{n_l}(x))||_{B_2} + ||K(f_{n_l}(x)) - K(f(x))||_{B_2}\
&leq& frac{varepsilon}{3} + frac{varepsilon}{3} + frac{varepsilon}{3} = varepsilon,
end{eqnarray*}$



whenever $l > N$ and $||y - x||_X < delta$, hence $(K circ f) in C(X,B_2)$. $square$




I'm stuck here, because I can't prove that $(K circ f) in C(X,B_2)$. I know that $(K circ f_{n_l}) in C(X,B_2)$ for each $l in mathbb{N}^*$, $(K circ f_{n_l})(x) rightarrow (K circ f)(x)$, $(K circ f_{n_l})$ is bounded I know the uniform limit of continuous functions is continuous, but I can't see how I can prove the pointwise convergence is, indeed, uniform. The only thing that I thought about this is use Ascoli's theorem because I know that $(K circ f_{n_l})$ is bounded, but I can't see how the family ${ (K circ f_{n_l}) in C(X,B_2) ; l in mathbb{N}^* }$ is equicontinuous.



I thought about the MaoWao's comment and I think I'm close to solving the question, but I don't sure if $(*)$ by the $N$ that I chose. If this is not the right $N$ that I need to take, someone can help me with a hint about how I can choose the $N$ without depending on $x$?



Thanks in advance!



$textbf{EDIT:}$



The version of Ascoli's theorem that I'm using:




$textbf{Ascoli-Arzela's theorem:}$ let be $E$ a set of continuous maps $f: K longrightarrow N$, where $K$ is compact. $E subset C(K,N)$ is relatively compact if, and only if, the following holds:



1) $E$ is equicontinuous;



2) For each $x in K$, $E(x) = { f(x) ; f in E }$ is relatively compact in $N$.











share|cite|improve this question











$endgroup$












  • $begingroup$
    The problem with your attempt is that the subsequence you choose depends on the point $x$. It is not immediate why there should be a subsequence working for all points simultaneously. Also, could you mention which version of Ascoli's theorem (if any) you already know.
    $endgroup$
    – MaoWao
    2 days ago












  • $begingroup$
    @MaoWao, I included the version of Ascoli's theorem that I have in mind.
    $endgroup$
    – George
    2 days ago










  • $begingroup$
    In your version of Ascoli-Arzela, what is $N$?
    $endgroup$
    – Nate Eldredge
    yesterday










  • $begingroup$
    @NateEldredge, $N$ is a metric space, this version of AScoli-Arzela is for metric spaces
    $endgroup$
    – George
    yesterday














1












1








1





$begingroup$


I'm studying by myself PDEs without having done Functional Analysis and I'm trying do the following exercise of the book "Partial Differential Equations I" by Michael E. Taylor on Appendix $A$ - Section $6$ (page $592$):





  1. Prove the following result, also known as part of Ascoli's theorem. If $X$ is a compact metric space, $B_j$ are Banach spaces, and $K: B_1 longrightarrow B_2$ is a compact operator, then $kappa f(x) = K(f(x))$ defines a compact map $kappa: mathcal{C}^{alpha}(X,B_1) longrightarrow C(X,B_2)$, for any $alpha > 0$.




$mathcal{C}^{alpha}$ denotes a space with $alpha$-Holder continuity (this notation was introduced on the final of the page $317$).



I would like to know if my attempt it's correct until where I wrote and, if it is correct, how I can ensure the limit below lives in $C(X,B_2)$.




$textbf{My attempt:}$



Firstly, we observe that $kappa$ is well-defined, otherwise, $K$ wouldn't well defined, which would be an absurd since $K$ is a map.



Now, given a bounded sequence $(f_n)$ in $mathcal{C}^{alpha}(X,B_1)$ and fixing $x in X$, we have that $(f_n(x))$ is a bounded sequence in $B_1$. By compactness of the map $K$, there is a subsequence $(f_{n_l}(x))$ in $B_1$ such that $K(f_{n_l}(x)) rightarrow K(f(x)) in B_2$, i.e., given $varepsilon > 0$, there is $N(x) in mathbb{N}$ such that



$$l > N(x) Longrightarrow ||K(f_{n_l}(x)) - K(f(x))||_{B_2} < frac{varepsilon }{3} (*)$$



The subsequence $((K circ f_{n_l}))$ is in $C(X,B_2)$ since $K$ and $f_{n_l}$ are continuous functions. Since the limit is unique, $f$ is well-defined.



By continuity of $K circ f_{n_l}$ in every $x in X$, there is $delta_x > 0$ such that



$$||y - x||_X < delta_x Longrightarrow ||K (f_{n_l})(y) - K (f_{n_l})(x)||_{B_2} < frac{varepsilon}{3} (**)$$



W.l.o.g., we assume $frac{1}{N(x)} < delta_x$. By compactness of $X$, there is a finite subcover of $X$ by open balls $B(x_i,frac{1}{N(x_i)}) subset X$, $i = 1, cdots, j$. Thus, arguing as before, we find $(K circ f_{n_l}(x))$ such that $(*)$ holds for $N := max_limits{ i in { 1,cdots,j } } N(x_i)$. Denoting by $delta := frac{1}{N}$ and using $(*)$ and $(**)$, we have that



$||K(f(y)) - K(f(x))||_{B_2} leq ||K(f(y)) - K(f_{n_l}(y))||_{B_2} + ||K(f_{n_l}(y)) - K(f_{n_l}(x))||_{B_2} + ||K(f_{n_l}(x)) - K(f(x))||_{B_2}$



$begin{eqnarray*}
||K(f(y)) - K(f(x))||_{B_2} &leq& ||K(f(y)) - K(f_{n_l}(y))||_{B_2} + ||K(f_{n_l}(y)) - K(f_{n_l}(x))||_{B_2} + ||K(f_{n_l}(x)) - K(f(x))||_{B_2}\
&leq& frac{varepsilon}{3} + frac{varepsilon}{3} + frac{varepsilon}{3} = varepsilon,
end{eqnarray*}$



whenever $l > N$ and $||y - x||_X < delta$, hence $(K circ f) in C(X,B_2)$. $square$




I'm stuck here, because I can't prove that $(K circ f) in C(X,B_2)$. I know that $(K circ f_{n_l}) in C(X,B_2)$ for each $l in mathbb{N}^*$, $(K circ f_{n_l})(x) rightarrow (K circ f)(x)$, $(K circ f_{n_l})$ is bounded I know the uniform limit of continuous functions is continuous, but I can't see how I can prove the pointwise convergence is, indeed, uniform. The only thing that I thought about this is use Ascoli's theorem because I know that $(K circ f_{n_l})$ is bounded, but I can't see how the family ${ (K circ f_{n_l}) in C(X,B_2) ; l in mathbb{N}^* }$ is equicontinuous.



I thought about the MaoWao's comment and I think I'm close to solving the question, but I don't sure if $(*)$ by the $N$ that I chose. If this is not the right $N$ that I need to take, someone can help me with a hint about how I can choose the $N$ without depending on $x$?



Thanks in advance!



$textbf{EDIT:}$



The version of Ascoli's theorem that I'm using:




$textbf{Ascoli-Arzela's theorem:}$ let be $E$ a set of continuous maps $f: K longrightarrow N$, where $K$ is compact. $E subset C(K,N)$ is relatively compact if, and only if, the following holds:



1) $E$ is equicontinuous;



2) For each $x in K$, $E(x) = { f(x) ; f in E }$ is relatively compact in $N$.











share|cite|improve this question











$endgroup$




I'm studying by myself PDEs without having done Functional Analysis and I'm trying do the following exercise of the book "Partial Differential Equations I" by Michael E. Taylor on Appendix $A$ - Section $6$ (page $592$):





  1. Prove the following result, also known as part of Ascoli's theorem. If $X$ is a compact metric space, $B_j$ are Banach spaces, and $K: B_1 longrightarrow B_2$ is a compact operator, then $kappa f(x) = K(f(x))$ defines a compact map $kappa: mathcal{C}^{alpha}(X,B_1) longrightarrow C(X,B_2)$, for any $alpha > 0$.




$mathcal{C}^{alpha}$ denotes a space with $alpha$-Holder continuity (this notation was introduced on the final of the page $317$).



I would like to know if my attempt it's correct until where I wrote and, if it is correct, how I can ensure the limit below lives in $C(X,B_2)$.




$textbf{My attempt:}$



Firstly, we observe that $kappa$ is well-defined, otherwise, $K$ wouldn't well defined, which would be an absurd since $K$ is a map.



Now, given a bounded sequence $(f_n)$ in $mathcal{C}^{alpha}(X,B_1)$ and fixing $x in X$, we have that $(f_n(x))$ is a bounded sequence in $B_1$. By compactness of the map $K$, there is a subsequence $(f_{n_l}(x))$ in $B_1$ such that $K(f_{n_l}(x)) rightarrow K(f(x)) in B_2$, i.e., given $varepsilon > 0$, there is $N(x) in mathbb{N}$ such that



$$l > N(x) Longrightarrow ||K(f_{n_l}(x)) - K(f(x))||_{B_2} < frac{varepsilon }{3} (*)$$



The subsequence $((K circ f_{n_l}))$ is in $C(X,B_2)$ since $K$ and $f_{n_l}$ are continuous functions. Since the limit is unique, $f$ is well-defined.



By continuity of $K circ f_{n_l}$ in every $x in X$, there is $delta_x > 0$ such that



$$||y - x||_X < delta_x Longrightarrow ||K (f_{n_l})(y) - K (f_{n_l})(x)||_{B_2} < frac{varepsilon}{3} (**)$$



W.l.o.g., we assume $frac{1}{N(x)} < delta_x$. By compactness of $X$, there is a finite subcover of $X$ by open balls $B(x_i,frac{1}{N(x_i)}) subset X$, $i = 1, cdots, j$. Thus, arguing as before, we find $(K circ f_{n_l}(x))$ such that $(*)$ holds for $N := max_limits{ i in { 1,cdots,j } } N(x_i)$. Denoting by $delta := frac{1}{N}$ and using $(*)$ and $(**)$, we have that



$||K(f(y)) - K(f(x))||_{B_2} leq ||K(f(y)) - K(f_{n_l}(y))||_{B_2} + ||K(f_{n_l}(y)) - K(f_{n_l}(x))||_{B_2} + ||K(f_{n_l}(x)) - K(f(x))||_{B_2}$



$begin{eqnarray*}
||K(f(y)) - K(f(x))||_{B_2} &leq& ||K(f(y)) - K(f_{n_l}(y))||_{B_2} + ||K(f_{n_l}(y)) - K(f_{n_l}(x))||_{B_2} + ||K(f_{n_l}(x)) - K(f(x))||_{B_2}\
&leq& frac{varepsilon}{3} + frac{varepsilon}{3} + frac{varepsilon}{3} = varepsilon,
end{eqnarray*}$



whenever $l > N$ and $||y - x||_X < delta$, hence $(K circ f) in C(X,B_2)$. $square$




I'm stuck here, because I can't prove that $(K circ f) in C(X,B_2)$. I know that $(K circ f_{n_l}) in C(X,B_2)$ for each $l in mathbb{N}^*$, $(K circ f_{n_l})(x) rightarrow (K circ f)(x)$, $(K circ f_{n_l})$ is bounded I know the uniform limit of continuous functions is continuous, but I can't see how I can prove the pointwise convergence is, indeed, uniform. The only thing that I thought about this is use Ascoli's theorem because I know that $(K circ f_{n_l})$ is bounded, but I can't see how the family ${ (K circ f_{n_l}) in C(X,B_2) ; l in mathbb{N}^* }$ is equicontinuous.



I thought about the MaoWao's comment and I think I'm close to solving the question, but I don't sure if $(*)$ by the $N$ that I chose. If this is not the right $N$ that I need to take, someone can help me with a hint about how I can choose the $N$ without depending on $x$?



Thanks in advance!



$textbf{EDIT:}$



The version of Ascoli's theorem that I'm using:




$textbf{Ascoli-Arzela's theorem:}$ let be $E$ a set of continuous maps $f: K longrightarrow N$, where $K$ is compact. $E subset C(K,N)$ is relatively compact if, and only if, the following holds:



1) $E$ is equicontinuous;



2) For each $x in K$, $E(x) = { f(x) ; f in E }$ is relatively compact in $N$.








functional-analysis proof-verification compact-operators






share|cite|improve this question















share|cite|improve this question













share|cite|improve this question




share|cite|improve this question








edited yesterday







George

















asked 2 days ago









GeorgeGeorge

838615




838615












  • $begingroup$
    The problem with your attempt is that the subsequence you choose depends on the point $x$. It is not immediate why there should be a subsequence working for all points simultaneously. Also, could you mention which version of Ascoli's theorem (if any) you already know.
    $endgroup$
    – MaoWao
    2 days ago












  • $begingroup$
    @MaoWao, I included the version of Ascoli's theorem that I have in mind.
    $endgroup$
    – George
    2 days ago










  • $begingroup$
    In your version of Ascoli-Arzela, what is $N$?
    $endgroup$
    – Nate Eldredge
    yesterday










  • $begingroup$
    @NateEldredge, $N$ is a metric space, this version of AScoli-Arzela is for metric spaces
    $endgroup$
    – George
    yesterday


















  • $begingroup$
    The problem with your attempt is that the subsequence you choose depends on the point $x$. It is not immediate why there should be a subsequence working for all points simultaneously. Also, could you mention which version of Ascoli's theorem (if any) you already know.
    $endgroup$
    – MaoWao
    2 days ago












  • $begingroup$
    @MaoWao, I included the version of Ascoli's theorem that I have in mind.
    $endgroup$
    – George
    2 days ago










  • $begingroup$
    In your version of Ascoli-Arzela, what is $N$?
    $endgroup$
    – Nate Eldredge
    yesterday










  • $begingroup$
    @NateEldredge, $N$ is a metric space, this version of AScoli-Arzela is for metric spaces
    $endgroup$
    – George
    yesterday
















$begingroup$
The problem with your attempt is that the subsequence you choose depends on the point $x$. It is not immediate why there should be a subsequence working for all points simultaneously. Also, could you mention which version of Ascoli's theorem (if any) you already know.
$endgroup$
– MaoWao
2 days ago






$begingroup$
The problem with your attempt is that the subsequence you choose depends on the point $x$. It is not immediate why there should be a subsequence working for all points simultaneously. Also, could you mention which version of Ascoli's theorem (if any) you already know.
$endgroup$
– MaoWao
2 days ago














$begingroup$
@MaoWao, I included the version of Ascoli's theorem that I have in mind.
$endgroup$
– George
2 days ago




$begingroup$
@MaoWao, I included the version of Ascoli's theorem that I have in mind.
$endgroup$
– George
2 days ago












$begingroup$
In your version of Ascoli-Arzela, what is $N$?
$endgroup$
– Nate Eldredge
yesterday




$begingroup$
In your version of Ascoli-Arzela, what is $N$?
$endgroup$
– Nate Eldredge
yesterday












$begingroup$
@NateEldredge, $N$ is a metric space, this version of AScoli-Arzela is for metric spaces
$endgroup$
– George
yesterday




$begingroup$
@NateEldredge, $N$ is a metric space, this version of AScoli-Arzela is for metric spaces
$endgroup$
– George
yesterday










1 Answer
1






active

oldest

votes


















2












$begingroup$

This will follow pretty directly from the version of Ascoli-Arzela that you already know.



Let $D$ be the unit ball of $C^alpha(X, B_1)$. Your goal is to show that $E = kappa D$ is relatively compact, i.e. that the set $E = {kappa f : f in D}$ is relatively compact in $C(X, B_2)$ (here $N = B_2$). You want to verify the hypotheses of Ascoli-Arzela.



For (1), use the Hölder continuity. You should be able to show that for any $f in D$ and any $x,y in X$, we have $|(kappa f)(x) - (kappa f)(y)|_{B_2} = |K(f(x)) - K(f(y))|_{B_2} le |K| d(x,y)^alpha$, where $|K|$ is the operator norm of $K$ and $d$ is the metric on $X$. Equicontinuity should then follow easily.



For (2), fix $x in X$ and note that for any $f in D$, we have $|f(x)|_{B_1} le 1$. Now use the fact that $K$ is a compact operator to conclude that ${(kappa f)(x) : f in D}$ is relatively compact in $B_2$.






share|cite|improve this answer









$endgroup$













  • $begingroup$
    I understood your hints, but I don't sure if I understand why you are considering $D$ as the unit ball of $mathcal{C}^{alpha}(X,B_1)$. The idea is consider $D$ as a bounded set, which is contained in a closed ball $overline{B}(0, A) subset mathcal{C}^{alpha}(X,B_1)$ for some $A > 0$. Since $f in D$, we must have $||f||_{C(X,B_2)} leq A$, then $||frac{1}{A} f||_{C(X,B_2)} leq 1$. Thus, we can suppose w.l.o.g. that $D = overline{B}(0,1)$, is this the reason why you assume $D$ the unit ball or the reason is more subtle?
    $endgroup$
    – George
    yesterday










  • $begingroup$
    Sorry if this seems a stupid question, but, as I said in the OP, I never did Functional Analysis, I just want to be sure if I understood the argument.
    $endgroup$
    – George
    yesterday










  • $begingroup$
    For me, the definition of "$kappa : Y_1 to Y_2$ is a compact operator" is "the image of the unit ball of $Y_1$ under $kappa$ is a relatively compact set in $Y_2$". What's your definition?
    $endgroup$
    – Nate Eldredge
    yesterday












  • $begingroup$
    The same of Appendix $D.5$ of Evans' book: "let $X$ and $Y$ be real Banach spaces. A bounded linear operator $$K: X longrightarrow Y$$ is called compact provided for each bounded sequence ${ u_k }_{k=1}^{infty} subset X$, the sequence ${ Ku_k }_{k=1}^{infty}$ is precompact in $Y$; that is, there exists a subsequence ${ u_{k_j} }_{j=1}^{infty}$ such that ${ Ku_{k_j} }_{j=1}^{infty}$ converges in $Y$."
    $endgroup$
    – George
    yesterday










  • $begingroup$
    Okay. Then the rescaling argument you gave above can be used to show that my definition is equivalent to Evans'.
    $endgroup$
    – Nate Eldredge
    yesterday













Your Answer





StackExchange.ifUsing("editor", function () {
return StackExchange.using("mathjaxEditing", function () {
StackExchange.MarkdownEditor.creationCallbacks.add(function (editor, postfix) {
StackExchange.mathjaxEditing.prepareWmdForMathJax(editor, postfix, [["$", "$"], ["\\(","\\)"]]);
});
});
}, "mathjax-editing");

StackExchange.ready(function() {
var channelOptions = {
tags: "".split(" "),
id: "69"
};
initTagRenderer("".split(" "), "".split(" "), channelOptions);

StackExchange.using("externalEditor", function() {
// Have to fire editor after snippets, if snippets enabled
if (StackExchange.settings.snippets.snippetsEnabled) {
StackExchange.using("snippets", function() {
createEditor();
});
}
else {
createEditor();
}
});

function createEditor() {
StackExchange.prepareEditor({
heartbeatType: 'answer',
autoActivateHeartbeat: false,
convertImagesToLinks: true,
noModals: true,
showLowRepImageUploadWarning: true,
reputationToPostImages: 10,
bindNavPrevention: true,
postfix: "",
imageUploader: {
brandingHtml: "Powered by u003ca class="icon-imgur-white" href="https://imgur.com/"u003eu003c/au003e",
contentPolicyHtml: "User contributions licensed under u003ca href="https://creativecommons.org/licenses/by-sa/3.0/"u003ecc by-sa 3.0 with attribution requiredu003c/au003e u003ca href="https://stackoverflow.com/legal/content-policy"u003e(content policy)u003c/au003e",
allowUrls: true
},
noCode: true, onDemand: true,
discardSelector: ".discard-answer"
,immediatelyShowMarkdownHelp:true
});


}
});














draft saved

draft discarded


















StackExchange.ready(
function () {
StackExchange.openid.initPostLogin('.new-post-login', 'https%3a%2f%2fmath.stackexchange.com%2fquestions%2f3138248%2fprove-that-the-following-map-is-compact%23new-answer', 'question_page');
}
);

Post as a guest















Required, but never shown

























1 Answer
1






active

oldest

votes








1 Answer
1






active

oldest

votes









active

oldest

votes






active

oldest

votes









2












$begingroup$

This will follow pretty directly from the version of Ascoli-Arzela that you already know.



Let $D$ be the unit ball of $C^alpha(X, B_1)$. Your goal is to show that $E = kappa D$ is relatively compact, i.e. that the set $E = {kappa f : f in D}$ is relatively compact in $C(X, B_2)$ (here $N = B_2$). You want to verify the hypotheses of Ascoli-Arzela.



For (1), use the Hölder continuity. You should be able to show that for any $f in D$ and any $x,y in X$, we have $|(kappa f)(x) - (kappa f)(y)|_{B_2} = |K(f(x)) - K(f(y))|_{B_2} le |K| d(x,y)^alpha$, where $|K|$ is the operator norm of $K$ and $d$ is the metric on $X$. Equicontinuity should then follow easily.



For (2), fix $x in X$ and note that for any $f in D$, we have $|f(x)|_{B_1} le 1$. Now use the fact that $K$ is a compact operator to conclude that ${(kappa f)(x) : f in D}$ is relatively compact in $B_2$.






share|cite|improve this answer









$endgroup$













  • $begingroup$
    I understood your hints, but I don't sure if I understand why you are considering $D$ as the unit ball of $mathcal{C}^{alpha}(X,B_1)$. The idea is consider $D$ as a bounded set, which is contained in a closed ball $overline{B}(0, A) subset mathcal{C}^{alpha}(X,B_1)$ for some $A > 0$. Since $f in D$, we must have $||f||_{C(X,B_2)} leq A$, then $||frac{1}{A} f||_{C(X,B_2)} leq 1$. Thus, we can suppose w.l.o.g. that $D = overline{B}(0,1)$, is this the reason why you assume $D$ the unit ball or the reason is more subtle?
    $endgroup$
    – George
    yesterday










  • $begingroup$
    Sorry if this seems a stupid question, but, as I said in the OP, I never did Functional Analysis, I just want to be sure if I understood the argument.
    $endgroup$
    – George
    yesterday










  • $begingroup$
    For me, the definition of "$kappa : Y_1 to Y_2$ is a compact operator" is "the image of the unit ball of $Y_1$ under $kappa$ is a relatively compact set in $Y_2$". What's your definition?
    $endgroup$
    – Nate Eldredge
    yesterday












  • $begingroup$
    The same of Appendix $D.5$ of Evans' book: "let $X$ and $Y$ be real Banach spaces. A bounded linear operator $$K: X longrightarrow Y$$ is called compact provided for each bounded sequence ${ u_k }_{k=1}^{infty} subset X$, the sequence ${ Ku_k }_{k=1}^{infty}$ is precompact in $Y$; that is, there exists a subsequence ${ u_{k_j} }_{j=1}^{infty}$ such that ${ Ku_{k_j} }_{j=1}^{infty}$ converges in $Y$."
    $endgroup$
    – George
    yesterday










  • $begingroup$
    Okay. Then the rescaling argument you gave above can be used to show that my definition is equivalent to Evans'.
    $endgroup$
    – Nate Eldredge
    yesterday


















2












$begingroup$

This will follow pretty directly from the version of Ascoli-Arzela that you already know.



Let $D$ be the unit ball of $C^alpha(X, B_1)$. Your goal is to show that $E = kappa D$ is relatively compact, i.e. that the set $E = {kappa f : f in D}$ is relatively compact in $C(X, B_2)$ (here $N = B_2$). You want to verify the hypotheses of Ascoli-Arzela.



For (1), use the Hölder continuity. You should be able to show that for any $f in D$ and any $x,y in X$, we have $|(kappa f)(x) - (kappa f)(y)|_{B_2} = |K(f(x)) - K(f(y))|_{B_2} le |K| d(x,y)^alpha$, where $|K|$ is the operator norm of $K$ and $d$ is the metric on $X$. Equicontinuity should then follow easily.



For (2), fix $x in X$ and note that for any $f in D$, we have $|f(x)|_{B_1} le 1$. Now use the fact that $K$ is a compact operator to conclude that ${(kappa f)(x) : f in D}$ is relatively compact in $B_2$.






share|cite|improve this answer









$endgroup$













  • $begingroup$
    I understood your hints, but I don't sure if I understand why you are considering $D$ as the unit ball of $mathcal{C}^{alpha}(X,B_1)$. The idea is consider $D$ as a bounded set, which is contained in a closed ball $overline{B}(0, A) subset mathcal{C}^{alpha}(X,B_1)$ for some $A > 0$. Since $f in D$, we must have $||f||_{C(X,B_2)} leq A$, then $||frac{1}{A} f||_{C(X,B_2)} leq 1$. Thus, we can suppose w.l.o.g. that $D = overline{B}(0,1)$, is this the reason why you assume $D$ the unit ball or the reason is more subtle?
    $endgroup$
    – George
    yesterday










  • $begingroup$
    Sorry if this seems a stupid question, but, as I said in the OP, I never did Functional Analysis, I just want to be sure if I understood the argument.
    $endgroup$
    – George
    yesterday










  • $begingroup$
    For me, the definition of "$kappa : Y_1 to Y_2$ is a compact operator" is "the image of the unit ball of $Y_1$ under $kappa$ is a relatively compact set in $Y_2$". What's your definition?
    $endgroup$
    – Nate Eldredge
    yesterday












  • $begingroup$
    The same of Appendix $D.5$ of Evans' book: "let $X$ and $Y$ be real Banach spaces. A bounded linear operator $$K: X longrightarrow Y$$ is called compact provided for each bounded sequence ${ u_k }_{k=1}^{infty} subset X$, the sequence ${ Ku_k }_{k=1}^{infty}$ is precompact in $Y$; that is, there exists a subsequence ${ u_{k_j} }_{j=1}^{infty}$ such that ${ Ku_{k_j} }_{j=1}^{infty}$ converges in $Y$."
    $endgroup$
    – George
    yesterday










  • $begingroup$
    Okay. Then the rescaling argument you gave above can be used to show that my definition is equivalent to Evans'.
    $endgroup$
    – Nate Eldredge
    yesterday
















2












2








2





$begingroup$

This will follow pretty directly from the version of Ascoli-Arzela that you already know.



Let $D$ be the unit ball of $C^alpha(X, B_1)$. Your goal is to show that $E = kappa D$ is relatively compact, i.e. that the set $E = {kappa f : f in D}$ is relatively compact in $C(X, B_2)$ (here $N = B_2$). You want to verify the hypotheses of Ascoli-Arzela.



For (1), use the Hölder continuity. You should be able to show that for any $f in D$ and any $x,y in X$, we have $|(kappa f)(x) - (kappa f)(y)|_{B_2} = |K(f(x)) - K(f(y))|_{B_2} le |K| d(x,y)^alpha$, where $|K|$ is the operator norm of $K$ and $d$ is the metric on $X$. Equicontinuity should then follow easily.



For (2), fix $x in X$ and note that for any $f in D$, we have $|f(x)|_{B_1} le 1$. Now use the fact that $K$ is a compact operator to conclude that ${(kappa f)(x) : f in D}$ is relatively compact in $B_2$.






share|cite|improve this answer









$endgroup$



This will follow pretty directly from the version of Ascoli-Arzela that you already know.



Let $D$ be the unit ball of $C^alpha(X, B_1)$. Your goal is to show that $E = kappa D$ is relatively compact, i.e. that the set $E = {kappa f : f in D}$ is relatively compact in $C(X, B_2)$ (here $N = B_2$). You want to verify the hypotheses of Ascoli-Arzela.



For (1), use the Hölder continuity. You should be able to show that for any $f in D$ and any $x,y in X$, we have $|(kappa f)(x) - (kappa f)(y)|_{B_2} = |K(f(x)) - K(f(y))|_{B_2} le |K| d(x,y)^alpha$, where $|K|$ is the operator norm of $K$ and $d$ is the metric on $X$. Equicontinuity should then follow easily.



For (2), fix $x in X$ and note that for any $f in D$, we have $|f(x)|_{B_1} le 1$. Now use the fact that $K$ is a compact operator to conclude that ${(kappa f)(x) : f in D}$ is relatively compact in $B_2$.







share|cite|improve this answer












share|cite|improve this answer



share|cite|improve this answer










answered yesterday









Nate EldredgeNate Eldredge

64.1k682174




64.1k682174












  • $begingroup$
    I understood your hints, but I don't sure if I understand why you are considering $D$ as the unit ball of $mathcal{C}^{alpha}(X,B_1)$. The idea is consider $D$ as a bounded set, which is contained in a closed ball $overline{B}(0, A) subset mathcal{C}^{alpha}(X,B_1)$ for some $A > 0$. Since $f in D$, we must have $||f||_{C(X,B_2)} leq A$, then $||frac{1}{A} f||_{C(X,B_2)} leq 1$. Thus, we can suppose w.l.o.g. that $D = overline{B}(0,1)$, is this the reason why you assume $D$ the unit ball or the reason is more subtle?
    $endgroup$
    – George
    yesterday










  • $begingroup$
    Sorry if this seems a stupid question, but, as I said in the OP, I never did Functional Analysis, I just want to be sure if I understood the argument.
    $endgroup$
    – George
    yesterday










  • $begingroup$
    For me, the definition of "$kappa : Y_1 to Y_2$ is a compact operator" is "the image of the unit ball of $Y_1$ under $kappa$ is a relatively compact set in $Y_2$". What's your definition?
    $endgroup$
    – Nate Eldredge
    yesterday












  • $begingroup$
    The same of Appendix $D.5$ of Evans' book: "let $X$ and $Y$ be real Banach spaces. A bounded linear operator $$K: X longrightarrow Y$$ is called compact provided for each bounded sequence ${ u_k }_{k=1}^{infty} subset X$, the sequence ${ Ku_k }_{k=1}^{infty}$ is precompact in $Y$; that is, there exists a subsequence ${ u_{k_j} }_{j=1}^{infty}$ such that ${ Ku_{k_j} }_{j=1}^{infty}$ converges in $Y$."
    $endgroup$
    – George
    yesterday










  • $begingroup$
    Okay. Then the rescaling argument you gave above can be used to show that my definition is equivalent to Evans'.
    $endgroup$
    – Nate Eldredge
    yesterday




















  • $begingroup$
    I understood your hints, but I don't sure if I understand why you are considering $D$ as the unit ball of $mathcal{C}^{alpha}(X,B_1)$. The idea is consider $D$ as a bounded set, which is contained in a closed ball $overline{B}(0, A) subset mathcal{C}^{alpha}(X,B_1)$ for some $A > 0$. Since $f in D$, we must have $||f||_{C(X,B_2)} leq A$, then $||frac{1}{A} f||_{C(X,B_2)} leq 1$. Thus, we can suppose w.l.o.g. that $D = overline{B}(0,1)$, is this the reason why you assume $D$ the unit ball or the reason is more subtle?
    $endgroup$
    – George
    yesterday










  • $begingroup$
    Sorry if this seems a stupid question, but, as I said in the OP, I never did Functional Analysis, I just want to be sure if I understood the argument.
    $endgroup$
    – George
    yesterday










  • $begingroup$
    For me, the definition of "$kappa : Y_1 to Y_2$ is a compact operator" is "the image of the unit ball of $Y_1$ under $kappa$ is a relatively compact set in $Y_2$". What's your definition?
    $endgroup$
    – Nate Eldredge
    yesterday












  • $begingroup$
    The same of Appendix $D.5$ of Evans' book: "let $X$ and $Y$ be real Banach spaces. A bounded linear operator $$K: X longrightarrow Y$$ is called compact provided for each bounded sequence ${ u_k }_{k=1}^{infty} subset X$, the sequence ${ Ku_k }_{k=1}^{infty}$ is precompact in $Y$; that is, there exists a subsequence ${ u_{k_j} }_{j=1}^{infty}$ such that ${ Ku_{k_j} }_{j=1}^{infty}$ converges in $Y$."
    $endgroup$
    – George
    yesterday










  • $begingroup$
    Okay. Then the rescaling argument you gave above can be used to show that my definition is equivalent to Evans'.
    $endgroup$
    – Nate Eldredge
    yesterday


















$begingroup$
I understood your hints, but I don't sure if I understand why you are considering $D$ as the unit ball of $mathcal{C}^{alpha}(X,B_1)$. The idea is consider $D$ as a bounded set, which is contained in a closed ball $overline{B}(0, A) subset mathcal{C}^{alpha}(X,B_1)$ for some $A > 0$. Since $f in D$, we must have $||f||_{C(X,B_2)} leq A$, then $||frac{1}{A} f||_{C(X,B_2)} leq 1$. Thus, we can suppose w.l.o.g. that $D = overline{B}(0,1)$, is this the reason why you assume $D$ the unit ball or the reason is more subtle?
$endgroup$
– George
yesterday




$begingroup$
I understood your hints, but I don't sure if I understand why you are considering $D$ as the unit ball of $mathcal{C}^{alpha}(X,B_1)$. The idea is consider $D$ as a bounded set, which is contained in a closed ball $overline{B}(0, A) subset mathcal{C}^{alpha}(X,B_1)$ for some $A > 0$. Since $f in D$, we must have $||f||_{C(X,B_2)} leq A$, then $||frac{1}{A} f||_{C(X,B_2)} leq 1$. Thus, we can suppose w.l.o.g. that $D = overline{B}(0,1)$, is this the reason why you assume $D$ the unit ball or the reason is more subtle?
$endgroup$
– George
yesterday












$begingroup$
Sorry if this seems a stupid question, but, as I said in the OP, I never did Functional Analysis, I just want to be sure if I understood the argument.
$endgroup$
– George
yesterday




$begingroup$
Sorry if this seems a stupid question, but, as I said in the OP, I never did Functional Analysis, I just want to be sure if I understood the argument.
$endgroup$
– George
yesterday












$begingroup$
For me, the definition of "$kappa : Y_1 to Y_2$ is a compact operator" is "the image of the unit ball of $Y_1$ under $kappa$ is a relatively compact set in $Y_2$". What's your definition?
$endgroup$
– Nate Eldredge
yesterday






$begingroup$
For me, the definition of "$kappa : Y_1 to Y_2$ is a compact operator" is "the image of the unit ball of $Y_1$ under $kappa$ is a relatively compact set in $Y_2$". What's your definition?
$endgroup$
– Nate Eldredge
yesterday














$begingroup$
The same of Appendix $D.5$ of Evans' book: "let $X$ and $Y$ be real Banach spaces. A bounded linear operator $$K: X longrightarrow Y$$ is called compact provided for each bounded sequence ${ u_k }_{k=1}^{infty} subset X$, the sequence ${ Ku_k }_{k=1}^{infty}$ is precompact in $Y$; that is, there exists a subsequence ${ u_{k_j} }_{j=1}^{infty}$ such that ${ Ku_{k_j} }_{j=1}^{infty}$ converges in $Y$."
$endgroup$
– George
yesterday




$begingroup$
The same of Appendix $D.5$ of Evans' book: "let $X$ and $Y$ be real Banach spaces. A bounded linear operator $$K: X longrightarrow Y$$ is called compact provided for each bounded sequence ${ u_k }_{k=1}^{infty} subset X$, the sequence ${ Ku_k }_{k=1}^{infty}$ is precompact in $Y$; that is, there exists a subsequence ${ u_{k_j} }_{j=1}^{infty}$ such that ${ Ku_{k_j} }_{j=1}^{infty}$ converges in $Y$."
$endgroup$
– George
yesterday












$begingroup$
Okay. Then the rescaling argument you gave above can be used to show that my definition is equivalent to Evans'.
$endgroup$
– Nate Eldredge
yesterday






$begingroup$
Okay. Then the rescaling argument you gave above can be used to show that my definition is equivalent to Evans'.
$endgroup$
– Nate Eldredge
yesterday




















draft saved

draft discarded




















































Thanks for contributing an answer to Mathematics Stack Exchange!


  • Please be sure to answer the question. Provide details and share your research!

But avoid



  • Asking for help, clarification, or responding to other answers.

  • Making statements based on opinion; back them up with references or personal experience.


Use MathJax to format equations. MathJax reference.


To learn more, see our tips on writing great answers.




draft saved


draft discarded














StackExchange.ready(
function () {
StackExchange.openid.initPostLogin('.new-post-login', 'https%3a%2f%2fmath.stackexchange.com%2fquestions%2f3138248%2fprove-that-the-following-map-is-compact%23new-answer', 'question_page');
}
);

Post as a guest















Required, but never shown





















































Required, but never shown














Required, but never shown












Required, but never shown







Required, but never shown

































Required, but never shown














Required, but never shown












Required, but never shown







Required, but never shown







Popular posts from this blog

Magento 2 - Add success message with knockout Planned maintenance scheduled April 23, 2019 at 23:30 UTC (7:30pm US/Eastern) Announcing the arrival of Valued Associate #679: Cesar Manara Unicorn Meta Zoo #1: Why another podcast?Success / Error message on ajax request$.widget is not a function when loading a homepage after add custom jQuery on custom themeHow can bind jQuery to current document in Magento 2 When template load by ajaxRedirect page using plugin in Magento 2Magento 2 - Update quantity and totals of cart page without page reload?Magento 2: Quote data not loaded on knockout checkoutMagento 2 : I need to change add to cart success message after adding product into cart through pluginMagento 2.2.5 How to add additional products to cart from new checkout step?Magento 2 Add error/success message with knockoutCan't validate Post Code on checkout page

Fil:Tokke komm.svg

Where did Arya get these scars? Unicorn Meta Zoo #1: Why another podcast? Announcing the arrival of Valued Associate #679: Cesar Manara Favourite questions and answers from the 1st quarter of 2019Why did Arya refuse to end it?Has the pronunciation of Arya Stark's name changed?Has Arya forgiven people?Why did Arya Stark lose her vision?Why can Arya still use the faces?Has the Narrow Sea become narrower?Does Arya Stark know how to make poisons outside of the House of Black and White?Why did Nymeria leave Arya?Why did Arya not kill the Lannister soldiers she encountered in the Riverlands?What is the current canonical age of Sansa, Bran and Arya Stark?